Cool Inequality 7-90 Followers Problem!

Algebra Level 5

a b ( a 2 b 2 ) + b c ( b 2 c 2 ) + c a ( c 2 a 2 ) M ( a 2 + b 2 + c 2 ) 2 \large{|ab\left( { a }^{ 2 }-{ b }^{ 2 } \right) +bc\left( { b }^{ 2 }-{ c }^{ 2 } \right) +ca\left( { c }^{ 2 }-{ a }^{ 2 } \right) |\le M{ \left( { a }^{ 2 }+{ b }^{ 2 }+{ c }^{ 2 } \right) }^{ 2 }}

For all real numbers a , b , c a,b,c the above inequality is satisfied. If the smallest value of constant M M can be expressed as

A B C \large{\frac { A }{ B } \sqrt { C } }

for some co-prime integers A A and B B and square-free number C C . Find A + B + C A+B+C


The answer is 43.

This section requires Javascript.
You are seeing this because something didn't load right. We suggest you, (a) try refreshing the page, (b) enabling javascript if it is disabled on your browser and, finally, (c) loading the non-javascript version of this page . We're sorry about the hassle.

2 solutions

Note that M^2 is the maximum of

( a b ( a 2 b 2 ) + a c ( c 2 a 2 ) + b c ( b 2 c 2 ) ) 2 ( a 2 + b 2 + c 2 ) 4 \frac{\left(a b \left(a^2-b^2\right)+a c \left(c^2-a^2\right)+b c \left(b^2-c^2\right)\right)^2}{\left(a^2+b^2+c^2\right)^4}

Letting { x = a + b + c , y = a b + a c + b c , a b c } \{x=a+b+c,y=a b+a c+b c,a b c\} , you can check that the above expression equals

x 2 ( 4 x 3 z + x 2 y 2 + 18 x y z 4 y 3 27 z 2 ) ( x 2 2 y ) 4 \frac{x^2 \left(-4 x^3 z+x^2 y^2+18 x y z-4 y^3-27 z^2\right)}{\left(x^2-2 y\right)^4}

I found this with a lot of computation (Mathematica). This is obviously not the best way to do it. Does anybody know of a nice way to figure this out without so much computation? Anyway, note that the above expression is quadratic in z. So maximizing this quadratic,we want to maximize

4 x 2 ( x 2 3 y ) 3 27 ( x 2 2 y ) 4 \frac{4 x^2 \left(x^2-3 y\right)^3}{27 \left(x^2-2 y\right)^4}

Setting Z = x 2 2 y Z=x^2-2 y :

4 ( Z y ) 3 ( 2 y + Z ) 27 Z 4 \frac{4 (Z-y)^3 (2 y+Z)}{27 Z^4}

This is a quartic in y, so using calculus, we find its maximum occurs at y = Z 8 y=-\frac{Z}{8} . Plugging this in yields M 2 = 81 512 M^2=\frac{81}{512} . Hence,

M = 9 16 2 M=\frac{9}{16 \sqrt{2}}

(Backtracking, we can easily show that this maximum can be achieved.)

This solution was very computation-heavy, so I welcome a better solution of any suggestions.

Shourya Pandey
Nov 9, 2015

IMO 2006, Problem 3( or so).

Please give a link for the solution

Department 8 - 5 years, 7 months ago

Log in to reply

Hint: a method is Lagrange Multipliers with some tweaking.

Daniel Prime - 5 years, 7 months ago

You can search the solutions on the website " www.imo.org". You can also search in google.

Priyanshu Mishra - 5 years, 6 months ago

Log in to reply

I know I have the solution but can you dare to right it here?

Department 8 - 5 years, 6 months ago

Log in to reply

@Department 8 Yes, sure. here is my solution.

We first consider the cubic polynomial

P ( t ) = t b ( t 2 b 2 ) + b c ( b 2 c 2 ) + c t ( c 2 t 2 ) P(t)=tb({ t }^{ 2 } - { b }^{ 2 }) + bc({ b }^{ 2 } - { c }^{ 2 }) + ct({ c }^{ 2 } - { t }^{ 2 }) .

It is easy to check that P ( b ) = P ( c ) = P ( b c ) = 0 P(b) = P(c) = P(-b - c) = 0 , and therefore

P ( t ) = ( b c ) ( t b ) ( t c ) ( t + b + c ) P(t) = (b - c)(t - b)(t - c)(t + b + c)

since the cubic coefficient is b c b - c . The left-hand side of the inequality can therefore be written in the form

a b ( a 2 b 2 ) + b c ( b 2 c 2 ) + c a ( c 2 a 2 ) = P ( a ) = ( b c ) ( a b ) ( a c ) ( a + b + c ) ab({ a }^{ 2 } - { b }^{ 2 }) + bc({ b }^{ 2 } - { c }^{ 2 }) + ca({ c }^{ 2 } - { a }^{ 2 })| = |P(a)| = |(b - c)(a - b)(a - c)(a + b + c)| . . . ( 1 ) ...(1)

Note that this expression is symmetric, and we can therefore assume a b c a ≤ b ≤ c without loss of generality. With this assumption,

|(a - b)(b - c)| = (b - a)(c - b) ≤ { \left( \frac { (b - a) + (c - b) }{ 2 } \right) }^{ 2 } = \frac { { (c - a) }^{ 2 } }{ 4 } , . . . ( 2 ) ...(2)

with equality if and only if b a = c b b - a = c - b , i.e. 2 b = a + c 2b = a + c . Also

{ \left( \frac { (c - b) + (b - a) }{ 2 } \right) }^{ 2 } \le \frac { { (c - b) }^{ 2 }{ + (b - a) }^{ 2 } }{ 2 } ,

or equivalently,

3 ( c a ) 2 2 [ ( b a ) 2 + ( c b ) 2 + ( c a ) 2 ] 3{ (c - a) }^{ 2 } ≤ 2·[{ (b - a) }^{ 2 } + { (c - b) }^{ 2 } + (c - a)^{ 2 }] , . . . ( 3 ) ...(3)

again with equality only for 2 b = a + c 2b = a + c . From ( 2 ) (2) and ( 3 ) (3) we get

( b c ) ( a b ) ( a c ) ( a + b + c ) |(b - c)(a - b)(a - c)(a + b + c)|

1 4 ( c a ) 3 ( a + b + c ) \le \frac { 1 }{ 4 } ·|(c-a)^{ 3 }(a+b+c) )

= 1 4 . ( c a ) 6 ( a + b + c ) 2 = \frac { 1 }{ 4 } .\sqrt { (c-a)^{ 6 }(a+b+c)^{ 2 } }

2 2 . ( ( 2. [ ( b a ) 2 + ( c a ) 2 + ( c b ) 2 ] 3 ) 3 . ( a + b + c ) 2 4 ) 2 \le \frac { \sqrt { 2 } }{ 2 } .{ \left( \sqrt [ 4 ]{ { \left( \frac { 2.[{ (b - a) }^{ 2 } + { (c - a) }^{ 2 } + { (c - b })^{ 2 }] }{ 3 } \right) }^{ 3 }.{ (a + b + c) }^{ 2 } } \right) }^{ 2 }

By the weighted AM-GM inequality this estimate continues as follows:

( b c ) ( a b ) ( a c ) ( a + b + c ) |(b - c)(a - b)(a - c)(a + b + c)|

9 2 32 . ( a 2 + b 2 + c 2 ) 2 \le \frac { 9\sqrt { 2 } }{ 32 } .{ ({ a }^{ 2 } + { b }^{ 2 } + { c }^{ 2 }) }^{ 2 } .

And hence M = 9 2 32 M = \boxed{\frac { 9\sqrt { 2 } }{ 32 }} .

Priyanshu Mishra - 5 years, 6 months ago

Log in to reply

@Priyanshu Mishra @Andrew Ellinor please convert this into a solution

Department 8 - 5 years, 6 months ago

Log in to reply

@Department 8 Yes, I was also seeking to write it in a solution column but did not find. So I replied you with the solution.

Priyanshu Mishra - 5 years, 6 months ago

0 pending reports

×

Problem Loading...

Note Loading...

Set Loading...